In right ΔDEF, DF = 20, m∠ F = 90˚, EF = 17. Which of the following is true? Does option 5 apply

In Right DEF, DF = 20, M F = 90, EF = 17. Which Of The Following Is True? Does Option 5 Apply

Answers

Answer 1

Answer:

Step-by-step explanation:

From the picture attached,

ΔDEF is a right triangle with two sides,

EF = 17 units

DF = 20 units

By applying Pythagoras theorem in the given triangle,

DE² = DF² + EF²

(20)² = DF² + (17)²

DF² = 400 - 289

DF = √111

Trigonometric ratios for the ∠F,

sin(F) = [tex]\frac{\text{Opposite side}}{\text{Hypotenuse}}[/tex]

[tex]\text{sin}F=\frac{\sqrt{111}}{20}[/tex]

[tex]\text{cosF}=\frac{\text{Adjacent side}}{\text{Hypotenuse}}[/tex]

[tex]\text{cos}F=\frac{17}{20}[/tex]

[tex]\text{tan}F=\frac{\text{Opposite side}}{\text{Adjacent side}}[/tex]

[tex]\text{tan}F=\frac{\sqrt{111}}{17}[/tex]

Choose the correct option.

In Right DEF, DF = 20, M F = 90, EF = 17. Which Of The Following Is True? Does Option 5 Apply

Related Questions

Christa needs to make a painting for art class.
She can only choose two of the eight colors listed
in the table above. What is the probability the two
colors she chooses are green and purple?

Answers

1/64
Because it is a
1/8 she gets green and a 1/8 she gets purple after so u multiply and get 1/64

Help is greatly appreciated:)

Answers

well there are 6 units down and 11 units across so 6•11=66. there are 14 red units. 66-14=52. the area of yellow is 52.

If f(x)=5x-3 find x=2

A.7
B.4
C.-4
D.-5

Answers

Answer: A. 7

Concept:

Here, we need to understand the idea of evaluation.  

When encountering questions that gave you an expression with variables, then stated: "If x = a, y = b, z = c" (a, b, c are all constants), this means you should substitute the value given for each variable back to the expression.

Solve:

Given Information

f(x) = 5x - 3

x = 2

Substitute the value into the function

f(2) = 5 (2) - 3

f(2) = 10 - 3

f(2) = 7

Hope this helps!! :)

Please let me know if you have any questions

What is the image of the point (-1,3) after a rotation of 270° counterclockwise
about the origin?

Answers

Answer:

3,-1

Step-by-step explanation:

The image of the point (-1,3) after a rotation of 270° counterclockwise about the origin is (3, -1).

The given coordinate point is (-1, 3).

What is a rotation of 270° counterclockwise about the origin?

When rotating a point 270 degrees counterclockwise about the origin our point A(x, y) becomes A'(y,-x). This means, we switch x and y and make x negative.

The point (-1, 3) is rotated 270° counterclockwise about the origin becomes (3, -1).

Therefore, the image of the point (-1,3) after a rotation of 270° counterclockwise about the origin is (3, -1).

To learn more about the rotation of 270° counterclockwise visit:

brainly.com/question/9109065.

#SPJ2

If x-6,x-2 and x+10 are three consecutive terms of a geometric progression,find the value of x​

Answers

Step-by-step explanation:

let the common ratio be r

(x-6)r=(x-2)

(x-2)r=(x+10)

r = (x-2)/(x-6)

and r = (x+10)/(x-2)

so, (x-2)/(x-6)=(x+10)/(x-2)

sloving it, you'll get x=8

so the value of x is 8

what is the difference of the fractions?
-2 1/2 - (-1 3/4)​

Answers

Answer:

-3/4

Step-by-step explanation:

First, let's convert the mixed numbers into improper fractions in an effort to make this problem easier to solve.

-2 1/2 as a mixed number is -5/2 and -1 3/4 as a mixed number is -7/4.

Our problem is now -5/2 - (-7/4).

We still can't solve this because the two fractions do not share a common denominator. 4 can serve as one, so -5/2 with a denominator of 4 would be -10/4.

The problem is now -10/4 - (-7/4).

Two negatives make a positive so the problem can be rewritten as -10/4 + 7/4. The final answer is -3/4.

Given two consecutive integers whose sum is 92, find the larger of the two integers.

Answers

Maybe i can help u (x+2)

PLEASE HELP QWQ AsAp with these 4 questions

Answers

Answer:

Step-by-step explanation:

I can't believe I'm doing this for 5 points, but ok!

For the first 3, we are going to multiply to find the value of that 3 x 3 matrix by picking up the first 2 columns and plopping them down at the end and then multiplying through using the rules for multiplying matrices:

[tex]\left[\begin{array}{ccccc}7&4&6&7&4\\-4&8&9&-4&8\\1&8&7&1&8\end{array}\right][/tex]  and from there find the sum of the products of the main axes minus the sum of the products of the minor axes, as follows (I'm not going to state the process in the next 2 problems, so make sure you follow it here. This is called the determinate. The determinate is what you get when you evaluate or find the value of a matrix. Just so you know):

[tex](7*8*7)+(4*9*1)+(6*-4*8)-[(1*8*6)+(8*9*7)+(7*-4*4)][/tex] which gives us:

392 + 36 - 192 - [48 + 504 - 112] which simplifies to

236 - 440 which is -204

On to the second one:

[tex]\left[\begin{array}{ccccc}-8&-4&-1&-8&-4\\1&7&-3&1&7\\8&9&9&8&9\end{array}\right][/tex] and multiplying gives us

[tex](-8*7*9)+(-4*-3*8)+(-1*1*9)-[(8*7*-1)+(9*-3*-8)+(9*1*-4)][/tex] which gives us:

-504 + 96 - 9 - [-56 + 216 - 36] which simplifies to

-417 - 124 which is -541, choice c.

Now for the third one:

[tex]\left[\begin{array}{ccccc}-2&-2&-5&-2&-2\\2&7&-3&2&7\\8&9&9&8&9\end{array}\right][/tex] and multiplying gives us

[tex](-2*7*9)+(-2*-3*8)+(-5*2*9)-[(8*7*-5)+(9*-3*-2)+(9*2*-2)][/tex] which gives us:

[tex]-126+48-90-[-280+54-36][/tex] which simplifies to

-168 - (-262) which is 94, choice c again.

Now for the last one. I'll show you the set up for the matrix equation; I solved it using the inverse matrix. So I'll also show you the inverse and how I found it.

[tex]\left[\begin{array}{cc}-4&-5&\\-6&-8\\\end{array}\right][/tex] [tex]\left[\begin{array}{c}x\\y\\\end{array}\right][/tex] = [tex]\left[\begin{array}{c}-5\\-2\\\end{array}\right][/tex] and I found the inverse of the 2 x 2 matrix on the left.

Find the inverse by:

* finding the determinate

* putting the determinate under a 1

* multiply that by the "mixed up matrix (you'll see...)

First things first, the determinate:

|A| = (-4*-8) - (-6*-5) which simplifies to

|A| = 32 - 30 so

|A| = 2; now put that under a 1 and multiply it by the mixed up matrix. The mixed up matrix is shown in the next step:

[tex]\frac{1}{2}\left[\begin{array}{cc}-8&5\\6&-4\end{array}\right][/tex]  (to get the mixed up matrix, swap the positions of the numbers on the main axis and then change the signs of the numbers on the minor axis). Now we multiply in the 1/2 to get the inverse:

[tex]\left[\begin{array}{cc}-4&\frac{5}{2}\\3&-2\\\end{array}\right][/tex] Multiply that inverse by both sides of the equation. This inverse "undoes" the matrix that's already there (like dividing the matrix that's already there by itself) which leaves us with just the matrix of x and y. Multiply the inverse matrix by the solution matrix:

[tex]\left[\begin{array}{c}x&y\end{array}\right] =\left[\begin{array}{cc}-4&\frac{5}{2} \\3&-2\end{array}\right] *\left[\begin{array}{c}-5&-2\\\end{array}\right][/tex] and that right side multiplies out to

x = 20 - 5 which is

x = 15 and

y = -15 + 4 which is

y = -11

(It works, I checked it)

What are the domain and range of f(x) = |x + 6|? Domain: (negative infinity, infinity); range: f(x) > 0 domain: x < -6; range: (negative infinity, infinity) domain: x > -6; range: (negative infinity, infinity) domain: (negative infinity, infinity) ; range: f(x) < 0

Answers

Answer:

Domain: [tex](-\infty,\infty)[/tex]

Range: [tex]f(x) \ge 0[/tex]

Step-by-step explanation:

Given

[tex]f(x) = |x + 6|[/tex]

Required

The domain and the range

First, we calculate the domain

[tex]f(x) = |x + 6|[/tex]

The above function does not have roots or fraction, where x is the denominator. This means that the domain is all real numbers, i.e. [tex](-\infty,\infty)[/tex]

The range

The function is an absolute function; So, the minimum value is 0.

Hence, the range is:

[tex]f(x) \ge 0[/tex]

Answer:

A

Step-by-step explanation:

on edge

Multiply the polynomial by distribution. Show your work and explain the steps you used to solve.
- 8x(x^2 – 8x + 3)

Answers

Answer:

-8x^3 + 64x^2 - 24x

Step-by-step explanation:

To distribute, we must multiple -8x to each term:

-8x(x^2) - 8x(-8x) - 8x(3)

-8x^3 + 64x^2 - 24x

Step-by-step explanation:

[tex] - 8x( {x}^{2} - 8x + 3) \\ = ( - 8x \times {x}^{2}) - ( - 8x \times 8x) + ( - 8x \times 3) \\ = - 8 {x}^{3} + 64 {x}^{2} - 24x[/tex]

distribute -8x to each term in the perenthesis i.e. multiply each term by -8x

Need helppppp asappppppp

Answers

Answer:

162

Step-by-step explanation:

because this is a parallelogram :

m<2 = m<4 so

4x - 22 = 3x - 12

4x - 3x = 22 - 12

x = 10 replace x with 10 in the equation for m<4

3x - 12 is 30 - 12 = 18

m<4 and m<1 are supplementary and their sum is equal to 180

m<4 + m<1 = 180

m<1 = 180 - 18

m<1 = 162

Miguel can use all or part of his $25 gift card to make a music purchase. Each song costs $1.50, and there is a $1.00 per account activation fee.

Which inequalities can represent this situation if m is the number of songs he can buy? Select two options.
1 + 1.5 m less-than-or-equal-to 25

Answers

Answer:

A, E or 1, 5

Step-by-step explanation:

Its not A, D

Find the equation of the line:
through (5, -1) and (2, 2).

Answers

Answer:

y = -x + 4

Step-by-step explanation:

(x₁,y₁)= (5, - 1)    & (x₂,y₂)= (2 , 2)

Slope = [tex]\frac{y_{2}-y_{1}}{x_{2}-x_{1}}[/tex]

         [tex]\frac{2-[-1]}{2-5}\\\\=\frac{2+1}{-3}\\\\=\frac{3}{-3}\\\\= -1[/tex]

m = -1

Equation of the line y - y₁ = m(x - x₁)

y  - [-1] = (-1)*(x - 5)

y + 1 = -x + 5      {-1 is distributed}

y = -x + 5 - 1

y = -x + 4

Which expressions are equivalent to
3(6+b) + 2b+1?
Select 2 answers.
Step 1 - Use the distributive property and pick your first
answer
Step 2 - Combine like terms and pick your second
answer

Answers

Step-by-step explanation:

3(6+b)+2b+1

18+3b+2b+1

5b+19

Once we distribute 3(6+b), 18 + 3b + 2b + 1. Our first answer is B. Next, we can combine like terms. This gives us 5b + 19. The other answer is C.

A lemonade recipe calls for 1/4 cups of lemon juice for every cup of water.

Plot the pairs in the table in a coordinate plane

Answers

Answer:

they are in order cross x with the y across from it

Step-by-step explanation:

A circle has a radius of 8cm. An angle of 1.4 radians is subtended at the center by an arc. Calculate the length of the arc

Answers

Answer:

11.2 cm

Step-by-step explanation:

Given that;

Arc Length Formula (if θ is in radians): l = ϴ × r

ϴ = angle subtended in radians

r= radius of the circle

l = 8cm × 1.4 radians

l= 11.2 cm

find the inequality represented by the graph​

Answers

Answer:

4x+3y<15

This is the inequality represented by the graph

What is the volume of this figure?

Answers

Step-by-step explanation:

3 x 3 x 3 = 27

2 x 2 x 2 = 8

6 x 10 x 2 = 120

27+8+120=155m³

A set of composite number less than 12.Express it in listing and set-builder methods

Answers

composite numbers less than 12={1,3,4,6,8,9,10}

Solve for $2\left(1-\frac{1}{2}\right)+3\left(1-\frac{1}{3}\right)+4\left(1-\frac{1}{4}\right)+\ldots
+10\left(1-\frac{1}{10}\right)=?$
Hall please

Answers

Answer:

[tex]$2\left(1-\frac{1}{2}\right)+3\left(1-\frac{1}{3}\right)+4\left(1-\frac{1}{4}\right)+\ldots+10\left(1-\frac{1}{10}\right)=?$[/tex]

[tex]2(\frac{2}{2} -\frac{1}{2} )+3(\frac{3}{3} -\frac{1}{3} )+4(\frac{4}{4} -\frac{1}{4} )+5(\frac{5}{5} -\frac{1}{5} )+6(\frac{6}{6}-\frac{1}{6} )+7(\frac{7}{7}-\frac{1}{7} )+8(\frac{8}{8} -\frac{1}{8} )+9(\frac{9}{9} -\frac{1}{9} )+10(\frac{10}{10} -\frac{1}{10} )\\\\=2(\frac{1}{2}) +3(\frac{2}{3})+4(\frac{3}{4})+5(\frac{4}{5}) +6(\frac{5}{6}) +7(\frac{6}{7}) +8(\frac{7}{8}) +9(\frac{8}{9}) +10(\frac{9}{10})\\\\=1+2+3+4+5+6+7+8+9\\\\=45[/tex]

if possible help me in this. I need to find the two odd numbers...​

Answers

Part (a)

Consecutive odd integers are integers that odd and they follow one right after another. If x is odd, then x+2 is the next odd integer

For example, if x = 7, then x+2 = 9 is right after.

Answer:  x+2

========================================================

Part (b)

The consecutive odd integers we're dealing with are x and x+2.

Their squares are x^2 and (x+2)^2, and these squares add to 394.

Answer: x^2 + (x+2)^2 = 394

========================================================

Part (c)

We'll solve the equation we just set up.

x^2 + (x+2)^2 = 394

x^2 + x^2 + 4x + 4 = 394

2x^2+4x+4-394 = 0

2x^2+4x-390 = 0

2(x^2 + 2x - 195) = 0

x^2 + 2x - 195 = 0

You could factor this, but the quadratic formula avoids trial and error.

Use a = 1, b = 2, c = -195 in the quadratic formula.

[tex]x = \frac{-b\pm\sqrt{b^2-4ac}}{2a}\\\\x = \frac{-(2)\pm\sqrt{(2)^2-4(1)(-195)}}{2(1)}\\\\x = \frac{-2\pm\sqrt{784}}{2}\\\\x = \frac{-2\pm28}{2}\\\\x = \frac{-2+28}{2} \ \text{ or } \ x = \frac{-2-28}{2}\\\\x = \frac{26}{2} \ \text{ or } \ x = \frac{-30}{2}\\\\x = 13 \ \text{ or } \ x = -15\\\\[/tex]

If x = 13, then x+2 = 13+2 = 15

Then note how x^2 + (x+2)^2 = 13^2 + 15^2 = 169 + 225 = 394

Or we could have x = -15 which leads to x+2 = -15+2 = -13

So, x^2 + (x+2)^2 = (-15)^2 + (-13)^2 = 225 + 169 = 394

We get the same thing either way.

Answer: Either 13, 15  or  -15, -13

plz do this all plz ​

Answers

Answer:

I don't know search it up I picked B

okay it's 5 and I'm

Brady scored a total of 320 points last season for his basketball team. This season, the team
added 2 extra games to the schedule. Brady thinks this will allow him to increase the total
number of points he scores by 5%. How many points is Brady expecting to score this season?

a. 336
b. 325
c. 304
d. 16

Answers

Brady is expecting 325 points this season.

Suppose a triangle has two sides of length 32 and 35, and that the angle
between these two sides is 120°. Which equation should you solve to find the
length of the third side of the triangle?
A. C2 = 322 + 352 – 2(32)(35)sin120°
B. sin32
sin35
b
120
C. C= 32 + 35 - 2(32)(35)cos120°
D. 2 = 322 + 352 - 2(32)(35)cos120°

Answers

Answer:

D

Step-by-step explanation:

It is the law of cos

C^2= A^2+B^2-2*A*B*cosL where L is the angle between A and B

so C = 32^2 + 35^2 - 2(32)(35)cos120°

3x + ky = 8
X – 2ky = 5
are simultaneous equations where k is a constant.
a) Show that x = 3.
b) Given that y = 1/2 determine the value of k.

Answers

Answer:

a) 3x + ky = 8

ky = 8 - 3x

x – 2ky = 5

x - 2(8 - 3x) = 5

x - 16 + 6x = 5

7x = 21

x = 3 (shown)

b) x-2ky = 5

sub x = 3, y = 1/2

3-2k(1/2) = 5

-2k = 2

k = -1

PLS HELP ME ON THIS QUESTION I WILL MARK YOU AS BRAINLIEST IF YOU KNOW THE ANSWER!!
Which of the following statements is correct about the data set 7, 10, 9, 100, 5, 12?
A. The data set has an outlier
B. The mean is the best and most accurate measure to describe this data set
C. The data set has two modes
D. The data set has a median of 10

Answers

Answer:

A. The data set has an outlier

Step-by-step explanation:

The data set has an outlier because 100 is the largest number of the set.

Answer:

option b

Step-by-step explanation:

Please Help and thank you so much!!

Answers

The probably of getting prime is 50%=0.5=1/2

which one is the right answers

Answers

Answer:

B

Step-by-step explanation:

n is the number of litres.

if you substitute 1 for 'n' then you will get :

C = 8 + 1.5(1)

C = $9.5

$9.50 for every litre

Answer:

C is correct

Step-by-step explanation:

The $8 charge is only once regardless of how many liters are ordered, so only answer C works.

Rewrite in simplest terms (-9x-2y)+(4x-5y)

Answers

Answer:

-5x - 7y

Step-by-step explanation:

-5x - 7y is the simplified answer to this question. Pls leave a thanks!

WILL GIVE BRAINLIEST PLS HELP

Answers

Answer:

Hello

Step-by-step explanation:

[tex]y=2x^2-12x+19\\=2(x^2-6x)+19\\=2(x^2-2*3*x+9)+19-18\\=2(x-3)^2+1\\\\Vertex\ is\ (3,1)\\\\Axis\ of\ symmetry\ is\ x=3\\\\y-intercept\ is \\\\y=2(0-3)^2+1=19\\[/tex]

Other Questions
Exercise 2.2.3: The cardinality of a power set. (a) What is the cardinality of P({1, 2, 3, 4, 5, 6}) Research and describe any recent theater production that reflects the political or social climate of the United States in the twenty-first century. Write your response in three or four paragraphs. Fix this grammatically incorrect sentence: The boats sails are very colorful. (There is ONLY ONE answer)A. The boats sails are very colorful.B. The boats sails are very colorful.C. The boats sails are very colorful.D. The boat sails are very colorful.E. The sails of the boat are colorful. Given their size and complexity, biological molecules such as carbohydrates, proteins, lipids, and nucleic acids can also be referred to as ____________. what's the name of contesting stage Which best explains why the Church was powerful? find from first principle the derivative of 3x+5/x help help help please please In the 1788 reading by Patrick Henry, he asserts that the foundation of America wasA. Free market economyB. Geographic Expansion C. Freedom of religion D. Individual liberty What percentage of 1hour is 6munites 20seconds Amelia, Luis, Shauna, and Clarence used different approaches to solve the inequality7.2b + 6.5 > 4.8b 8.1.Amelia started by subtracting 7.2b from both sides to get 6.5 > 2.4b 8.1.Luis started by subtracting 4.8b from both sides to get 2.4b + 6.5 < 8.1.Shauna started by subtracting 6.5 from both sides to get 7.2b > 4.8b 14.6.Clarence started by adding 8.1 to both sides to get 7.2b + 14.6 > 4.8b.Which students first step was incorrect, and why?Amelias, because the variable term must be isolated on the left sideLuiss, because he flipped the inequality sign when he subtractedShaunas, because she did not apply the subtraction property of equality properly Clarences, because the terms he added together were not like terms When CP = Rs 1250 and SP = Rs 1500, find profit or loss percent ? The phylogenetic tree above represents the three domains of organisms on our planet. According to this tree, organisms in domain Eukarya are closer related to organisms in domain Archaea. Organisms in kingdom Eukarya can be distinguished from organisms in kingdom Archaea by which of the following distinctions?Group of answer choicesThe presence of ribosomes for protein synthesis.The use of a cell wall for protection and supportThe presence of a nucleus in their cellsThe storage of genetic material in DNA molecules. Which invention did not lead to industrial growth?A. Electricity B. Telephone ServicesC. Advertising What were the origins of slavery What is the solution for this equation: Qstns:-1) is Japan a developed country?2) Is Japan rich?3) Is Japan clean?4) Is Japan really advanced?5) For what technologies is Japan famous for? ^O^ Matthew participates in a study that is looking at how confident students at SUNY Albany are. The mean score on the scale is 50. The distribution has a standard deviation of 10 and is normally distributed. Matthew scores a 65. What percentage of people could be expected to score the same as Matthew or higher on this scale?a) 93.32%b) 6.68%c) 0.07%d) 43.32% An online retailer is looking to implement an enterprise platform. Which component of the enterprise platform will help the company capture, curate, and consume customer information to improve their services? As a child, Eric once told his mother that he thought their house would look pretty if it was on fire. His mother immediately scolded Eric, screaming that he should not think such things, and then sent him to his room without dinner. Eric had repeated and consistent experiences like this when he would express an idea and his mother would harshly scold him as if having such a thought might lead to the event happening. This type of experience may lead to a psychological vulnerability to develop OCD called: